LSAT and Law School Admissions Forum

Get expert LSAT preparation and law school admissions advice from PowerScore Test Preparation.

 Adam Tyson
PowerScore Staff
  • PowerScore Staff
  • Posts: 5153
  • Joined: Apr 14, 2011
|
#74304
I assume you mean KMQL, gagomez? The problem with that solution is that it breaks two rules: we are required to include exactly one of N or P, and neither is included, and we are required to pick exactly two of LMQ, but this solution includes all three of them. That's why in our explanation we said that selecting M and Q forces L out (which in turn forces N out, which then forces P in).
User avatar
 Onohitae
  • Posts: 1
  • Joined: Mar 10, 2023
|
#99474
For this question, I eliminated every answer containing L or K because if those are selected then the sufficient condition can be placed anywhere depending on the other rules. I did not choose E immediately but I started from there to check if it was correct. Is this a good way to go about this question?
 Robert Carroll
PowerScore Staff
  • PowerScore Staff
  • Posts: 1787
  • Joined: Dec 06, 2013
|
#99480
Onohitae,

That seems a really good approach to me, actually! When those necessary conditions are met (L or K, as you said), the sufficient conditions are now more free to be in or out. That's not very restrictive, so it's unlikely to completely determine the assignment of diners. Exactly that kind of thinking will save you time for questions like this - well done!

Robert Carroll

Get the most out of your LSAT Prep Plus subscription.

Analyze and track your performance with our Testing and Analytics Package.